LSAT and Law School Admissions Forum

Get expert LSAT preparation and law school admissions advice from PowerScore Test Preparation.

 Administrator
PowerScore Staff
  • PowerScore Staff
  • Posts: 8916
  • Joined: Feb 02, 2011
|
#39180
Complete Question Explanation
(The complete setup for this game can be found here: lsat/viewtopic.php?t=15163)

The correct answer choice is (E)

If W is third, we need to work with Template 1A in determining the order of the remaining variables (no other solution allows for W to be third):
PT74 - Game_#2_#8_diagram 1.png
To answer this Could Be True question most efficiently, focus on the uncertainty in this solution: the correct answer choice is most likely to involve either G or J, which helps eliminate answer choices (A) and (D).

Answer choice (A) is incorrect, because F gives the S lecture, not the W lecture.

Answer choice (B) is incorrect, because the O lecture is given by H, not G.

Answer choice (C) is incorrect, because the S lecture is given by F, not G.

Answer choice (D) is incorrect, because H gives the O lecture, not the S lecture.

Answer choice (E) is the correct answer choice, because J could give the L lecture.
You do not have the required permissions to view the files attached to this post.

Get the most out of your LSAT Prep Plus subscription.

Analyze and track your performance with our Testing and Analytics Package.